Đến nội dung

longatk08 nội dung

Có 348 mục bởi longatk08 (Tìm giới hạn từ 30-05-2020)



Sắp theo                Sắp xếp  

#598289 $\sum_{cyc}\frac{a^3}{a+b}\...

Đã gửi bởi longatk08 on 14-11-2015 - 16:26 trong Bất đẳng thức - Cực trị

Cho $a,b,c$ không âm thỏa $ab+bc+ac>0$.Chứng minh rằng:

 

$$\frac{a^3}{a+b}+\frac{b^3}{b+c}+\frac{c^3}{c+a}\leq (1+\frac{\sqrt[3]{4}}{3}).(\frac{a^3+b^3+c^3}{a+b+c})$$




#597744 $\frac{a}{ab+3}+\frac{b}{bc...

Đã gửi bởi longatk08 on 10-11-2015 - 21:06 trong Bất đẳng thức - Cực trị

Cho $a,b,c$ không âm thỏa mãn $a^2+b^2+c^2=3$.Chứng minh rằng:

 

$$\frac{a}{ab+3}+\frac{b}{bc+3}+\frac{c}{ca+3}\leq \frac{3}{4}$$




#597742 $\sum \frac{2a}{2a+b+c}\geq 1+\f...

Đã gửi bởi longatk08 on 10-11-2015 - 21:01 trong Bất đẳng thức - Cực trị

 $(a,b,c)=(1,1,5)$ ?

Ý tưởng lớn gặp nhau chăng :)




#597741 $\sum \frac{2a}{2a+b+c}\geq 1+\f...

Đã gửi bởi longatk08 on 10-11-2015 - 21:00 trong Bất đẳng thức - Cực trị

Cho $a,b,c>0$. C/m:
$2(a+b+c)(ab+bc+ca)^{2}\geq 9abc(a^{2}+b^{2}+c^{2}+ab+ca+bc)$

BĐT trên sau khi khai triển: $2\sum a^{2}b^{2}(a+b)+abc(a+b+c)\geq5abc(a^{2}+b^{2}+c^{2})$

Lấy $b=c=1, a=5$ sai




#597244 $\frac{1}{a+b}+\frac{1}{c+b...

Đã gửi bởi longatk08 on 07-11-2015 - 16:49 trong Bất đẳng thức và cực trị

Với $a,b,c \geq 0$, $ab+bc+ac=1$.Ta có các kết quả tương tự sau:

 

$$\frac{1}{a+b}+\frac{1}{b+c}+\frac{1}{c+a}\geq 2+\frac{1}{a+b+c}$$

 

$$\frac{1}{a+b}+\frac{1}{b+c}+\frac{1}{c+a}+\frac{3}{a+b+c}\geq 4$$

 

$$ \frac{1}{a^2+b^2}+\frac{1}{b^2+c^2}+\frac{1}{c^2+a^2}\geq \frac{5}{2}$$




#597181 Cho a,b,c $\geq$ 0 , ab+bc+ca=1

Đã gửi bởi longatk08 on 06-11-2015 - 23:34 trong Bất đẳng thức - Cực trị

 

Cho a+b+c $\geq$ 0 , ab+bc+ca=1. Chứng minh rằng:

   \[{\left( {\frac{{{a^2}}}{{a + c}} + \frac{{{b^2}}}{{a + b}} + \frac{{{c^2}}}{{c + b}}} \right)^2} + \frac{{4abc({a^2} + {b^2} + {c^2} + 2)}}{{(a + b)(b + c)(c + a)}} \ge \frac{9}{4}\]

 

Lấy $a+b+c=1$ và đặt $ab+bc+ac=q,r=abc$ thì ta cần chứng minh:

 

$$\frac{(q-q^2-2r)^2}{(q-r)^2}+\frac{4r}{q-r}\geq \frac{9q}{4}$$

 

Khai triển thì $f(r)$ là hàm lồi theo $r$ nên có cực đại.

 

BĐT tương đương: $9qr^2-34q^2r+17q^3-4q^4-4q^2 \leq 0$

 

Nếu $q \leq \frac{1}{4}$ thì $r \geq 0$. Nếu $\frac{1}{3} \geq q \geq \frac{1}{4}$ thì $r \geq \frac{4q-1}{9}$ và $f(\frac{4q-1}{9})=\frac{q(1-3q)^2(1-4q)}{9}\leq 0$

 

Xét $r \leq \frac{q^2}{3}$ thì $f(\frac{q^2}{3})=\frac{q^2(3q-1)(q^2-15q+12)}{3}\leq 0$




#596892 $\sum \frac{1}{1+a^2(b+c)}$$...

Đã gửi bởi longatk08 on 04-11-2015 - 23:43 trong Bất đẳng thức và cực trị

BĐT sau mạnh hơn:

 

$$\frac{1}{1+a^2(b+c)}+\frac{1}{1+b^2(a+c)}+\frac{1}{1+c^2(a+b)}\leq \frac{3}{1+2abc}$$




#596890 $\sum \frac{1}{1+a^2(b+c)}$$...

Đã gửi bởi longatk08 on 04-11-2015 - 23:35 trong Bất đẳng thức và cực trị

BĐT tương tự với $a+b+c=3$, $a,b,c$ không âm:

 

$$\frac{a}{1+(b+c)^2}+\frac{b}{1+(a+c)^2}+\frac{c}{1+(a+b)^2}\leq \frac{60}{1+99abc}$$




#596532 $ \dfrac{b+c}{2a^2+bc} + \dfrac{a+c}{2 b^2+ca } + \dfrac...

Đã gửi bởi longatk08 on 02-11-2015 - 00:06 trong Bất đẳng thức và cực trị

mình có bài bdt này đã có đề từ lâu mà chưa biêt cách giai
a,b,c>0. Cm
$ \dfrac{b+c}{2a^2+bc} + \dfrac{a+c}{2 b^2+ca } + \dfrac{a+b}{2c^2+ab } \geq \dfrac{6}{a+b+c}$

Cho các số thực không âm $a,b,c$ thỏa mãn $ab+bc+ac>0$.Chứng minh rằng:

 

$$\frac{a+b+c}{ab+bc+ac}\geq \frac{a}{a^2+2bc}+\frac{b}{b^2+2ac}+\frac{c}{c^2+2ab}+\frac{8abc(a^2+b^2+c^2-ab-bc-ac)}{(a+b+c)^4(ab+bc+ac)}$$




#596140 $a^{3}+b^{3}+c^{3}+3abc\geq a^{2...

Đã gửi bởi longatk08 on 30-10-2015 - 20:24 trong Bất đẳng thức và cực trị

 Cho $a,b,c$ là các số thực không âm sao cho không có bất kì 2 trong 3 số đồng thời bằng 0. Chứng minh rằng :

$$a^3+b^3+c^3+3abc.\dfrac{a^2b+b^2c+c^2a}{ab^2+bc^2+ba^2}\geq \sum ab(a+b)$$

 

 BW too :luoi:

Cái này phân tích bình phương hoán vị thì tốt hơn em ạ.




#596050 $a^{3}+b^{3}+c^{3}+3abc\geq a^{2...

Đã gửi bởi longatk08 on 30-10-2015 - 00:13 trong Bất đẳng thức và cực trị

Cho $a,b,c$ không âm thỏa $ab+bc+ac>0$.Chứng minh BĐT sau:

 

$$ \sum a(a-b)(a-c)\geq abc(\frac{a^2+b^2+c^2}{ab+bc+ca}-1)$$




#596049 $a^{3}+b^{3}+c^{3}+3abc\geq a^{2...

Đã gửi bởi longatk08 on 30-10-2015 - 00:09 trong Bất đẳng thức và cực trị

Cách giải khác không dùng BĐT $Schur$  

Giả sử $a \geq b \geq c$ 

Đặt $x=a-b,y=b-c$

Bất đẳng thức được viết lại thành

$c(x+y)y-(c+y)xy+(c+x+y)x(x+y) \geq 0$

$<=>c(x^{2}+xy+y^{2})+x^{2}(x+2y) \geq 0$ ( hiển nhiên đúng vì $c;x;y$ không âm )

Dấu bằng xảy ra khi $x=y=0$ hoặc $x=c=0$

Hay $a=b=c$ hoặc $a=b,c=0$ và các hoán vị

Cho $a,b,c$ không âm thỏa mãn không có 2 số nào đồng thời bằng không.Chứng minh BĐT sau:

 

$$\sum a^2(a-b)(a-c)\geq \frac{(a+b-c)(b+c-a)(c+a-b)}{ab+bc+ca}\cdot \sum a(a-b)(a-c)$$

 

$$ \sum a(a-b)(a-c)\geq 4\frac{(a-b)^2(b-c)^2(c-a)^2}{(a+b)(a+c)(b+c)}$$

 

BW are welcome! Have fun...




#595773 Đề thi học sinh giỏi tỉnh Bình Thuận năm 2015-2016 ( vòng 2 )

Đã gửi bởi longatk08 on 28-10-2015 - 16:04 trong Thi HSG cấp Tỉnh, Thành phố. Olympic 30-4. Đề thi và kiểm tra đội tuyển các cấp.

Bài bất dùng bổ đề:

 

$$\frac{x}{y}+\frac{y}{z}+\frac{z}{x}\geq \frac{x+y+z}{\sqrt[3]{xyz}}$$

 

Vế sau là BĐT trong đề VMO năm 96




#595547 $\sum \frac{x^2+1}{x^2+2yz+1}\geq 2...

Đã gửi bởi longatk08 on 26-10-2015 - 23:04 trong Bất đẳng thức - Cực trị

Cách của anh Huyện thì em biết vì đây cũng là lời giải ban đầu của em, em vẫn đang tìm 1 lời giải khác bằng C-S  :D




#595373 Mới chế bài BĐT : $27abc \geq 15(ab+ac+bc) - 4$

Đã gửi bởi longatk08 on 25-10-2015 - 22:08 trong Bất đẳng thức và cực trị

BĐT tương tự sau, tuy không quá chặt nhưng coi như món "quà" dành cho chủ thớt:

 

Cho $a,b,c$ thực dương và $a+b+c=3$.Chứng minh rằng:

 

$$a^3b+b^3c+c^3a +6abc \leq 9$$




#595365 Mới chế bài BĐT : $27abc \geq 15(ab+ac+bc) - 4$

Đã gửi bởi longatk08 on 25-10-2015 - 21:58 trong Bất đẳng thức và cực trị

Tại vì em chứng minh nó dài dòng lắm ạ, ai chứng minh gọn em phục ạ

Nói thật thì bài này còn yếu hơn cả BĐT Schur mà ta vẫn hay dùng và cũng chả đáng để dùng thêm bất cứ 1 công cụ nào khác ở đây ngoài Schur và AM-GM

 

Sau khi đồng bậc 2 vế và rút gọn thì ta có BĐT tương đương:

 

$4(a^3+b^3+c^3)+6abc \geq 3ab(a+b)+3bc(b+c)+3ac(a+c)$

 

Nhân tiện đây thì bạn nên đọc kĩ nội quy trc khi post bài. Thanks.




#595361 Mới chế bài BĐT : $27abc \geq 15(ab+ac+bc) - 4$

Đã gửi bởi longatk08 on 25-10-2015 - 21:52 trong Bất đẳng thức và cực trị

Bất đẳng thức sau do mình chế ra, được đánh giá là rất mạnh, mong cao thủ chỉ giáo ạ 

 

Cho 3 số dương a,b,c thỏa :  a+b+c = 1

 

cmr :

$27abc \geq 15(ab+ac+bc) - 4$

Spoiler

 

Tại sao bạn nghĩ bài này mạnh vậy?




#595177 $\sum \frac{x^2+1}{x^2+2yz+1}\geq 2...

Đã gửi bởi longatk08 on 24-10-2015 - 23:09 trong Bất đẳng thức - Cực trị

Cho $x,y,z$ thực dương thỏa mãn $xy+yz+xz=1$. Chứng minh rằng:

 

$$\frac{x^2+1}{x^2+2yz+1}+\frac{y^2+1}{y^2+2xz+1}+\frac{z^2+1}{z^2+2xy+1}\geq 2$$

 

Spoiler




#595140 $\sum \frac{(x+y)^2}{x^2+y^2+xy+yz}\l...

Đã gửi bởi longatk08 on 24-10-2015 - 20:28 trong Bất đẳng thức - Cực trị

ta có $\frac{(x+y)^2}{x^2+y^2+xy+yz}\leq \frac{x^2}{x^2+xy}+\frac{y^2}{y^2+yz}=\frac{x}{x+y}+\frac{y}{y+z}$  (svax)

mấy cái tê tương tự rồi cộng lại

Các phân thức sau ta vẫn thu được đại lượng này hay nói cách khác cách làm này sai.




#595139 $\sum \sqrt{\dfrac{a}{b+c}}...

Đã gửi bởi longatk08 on 24-10-2015 - 20:26 trong Bất đẳng thức và cực trị

em không biết ý kiến của em có vấn đề không :))
ta có $\sum \sqrt{\frac{a}{b+c}} \geq 2$, dấu bằng tại a=b,c=0, thay vào thì lại là 10 ạ
có gì sai mong anh lượng thứ ^^

Tại $c=0$ xảy ra dấu bằng, cái chính là xác định xem $a=kb$ ,$k$ là bao nhiêu? chứ ra 10 thì ta vẫn có dấu ">"




#595050 $\sum \frac{(x+y)^2}{x^2+y^2+xy+yz}\l...

Đã gửi bởi longatk08 on 23-10-2015 - 23:37 trong Bất đẳng thức - Cực trị

Cho $x,y,z$ thực dương.Chứng minh BĐT sau:

 

$$\frac{(x+y)^2}{x^2+y^2+xy+yz}+\frac{(y+z)^2}{y^2+z^2+yz+zx}+\frac{(z+x)^2}{z^2+x^2+xy+xz}\leq 3$$

 




#595042 $\sum \sqrt{\dfrac{a}{b+c}}...

Đã gửi bởi longatk08 on 23-10-2015 - 23:01 trong Bất đẳng thức và cực trị

Cho $a,b,c$ không âm thỏa $ab+bc+ac>0$.Chứng minh rằng:

 

$$\sqrt{\frac{a}{b+c}}+\sqrt{\frac{b}{a+c}}+\sqrt{\frac{c}{a+b}}+\frac{2(a+b+c)^3}{a^3+b^3+c^3}\geq 6$$




#595040 $\int \frac{1}{(1+\sqrt{2x+1})^2}dx$

Đã gửi bởi longatk08 on 23-10-2015 - 22:52 trong Tích phân - Nguyên hàm

$\int \frac{1}{(1+\sqrt{2x+1})^2}dx$

Đổi biến số rồi vi phân. Đặt $\sqrt{2x+1}=t$




#594988 $P=\frac{x-1}{y^2}+\frac{y-1}...

Đã gửi bởi longatk08 on 23-10-2015 - 18:13 trong Bất đẳng thức và cực trị

Cho các số thực x,y,z thỏa mãn $x+y+z=xyz$ và $x>1, y>1, z>1$. Tìm GTNN của 

$P=\frac{x-1}{y^2}+\frac{y-1}{z^2}+\frac{z-1}{x^2}$

Sau khi đổi biến thì ta sẽ đi chứng minh rằng với $a,b,c$ thực dương thỏa $ab+bc+ac=1$ thì ta có:

 

$$P=\frac{a^2}{b}+\frac{b^2}{c}+\frac{c^2}{a}-a^2-b^2-c^2\geq \sqrt{3}-1$$

 

Hãy chứng minh BĐT sau:

 

$$\frac{a^2}{b}+\frac{b^2}{c}+\frac{c^2}{a}\geq\frac{(a^2+b^2+c^2)(a+b+c)}{ab+bc+ac}$$




#594783 $5(ab+bc+ca)(a+b+c)\geq (a+b+c)^{3}+18abc$

Đã gửi bởi longatk08 on 21-10-2015 - 23:37 trong Bất đẳng thức - Cực trị

Cho a, b, c là 3 cạnh của 1 tam giác. CMR

$5(ab+bc+ca)(a+b+c)\geq (a+b+c)^{3}+18abc$

Dùng phép thế Ravi rồi khai triển ra.